Page 1 of 1

True or false

Posted: Sat Feb 04, 2017 5:46 pm
by Papapetros Vaggelis
Examine if the following is true.

If \(\displaystyle{p}\) is a prime number and\(\displaystyle{\left(G,\cdot\right)}\) is a group

such that \(\displaystyle{o(G)=p^3}\) and \(\displaystyle{x^p=1\,,\forall\,x\in G}\) then

\(\displaystyle{\left(G,\cdot\right)}\) is abelian.